Интеграл с двумя энергетическими функциями Грина

Проблема

Я пытаюсь оценить этот интеграл:

я ( к ) "=" лим ϵ 0 г 3 д 1 Е Е д + я ϵ 1 Е Е к + д + я ϵ

с Е д "=" д 2 / 2 и Е к + д "=" | к + д | 2 / 2 .

Если возможно, было бы неплохо увидеть несколько способов решения.

Моя попытка

Я решил работать в сферической системе координат с к выровнены вдоль г -ось. Сдача ф представлять полярный угол между к и д , Я написал Е к + д как

Е к + д "=" 1 2 ( к 2 + д 2 2 к д потому что ( π ф ) ) "=" к 2 2 + д 2 2 + к д потому что ф .

Тогда довольно просто вычислить оба угловых интеграла, оставив только радиальный интеграл:

я ( к ) "=" лим ϵ 0 2 π к 0 г д д Е д 2 / 2 + я ϵ п | Е ( к д ) 2 / 2 + я ϵ Е ( к + д ) 2 / 2 + я ϵ | .

Но тут я застрял.

Хотя я согласен с тем, что здесь это не совсем неуместно, поскольку здесь речь идет об интегралах, встречающихся в теории поля, я думаю, что это больше относится к Math.SE. Я лично задавал там подобные вопросы и мне повезло!
Спасибо за комментарий! Должен ли я удалить этот вопрос и повторно задать его там? Или спросить его там и связать их как-то? (Извините, новичок в этом.)
Честно говоря, я тоже новичок в этом. Я видел , как вопросы перемещались, хотя я не знаю, как это происходит, возможно, это делают модераторы. В любом случае, не помешает задать его там, так как я подозреваю, что этот вопрос, вероятно, все равно будет отмечен здесь :) Но вам не нужно удалять его, если вы этого не хотите. Кроме того, я подозреваю, что вы имели в виду к ± д в последнем уравнении.
Спасибо, и вы правы насчет опечатки, хороший улов! Починил это.

Ответы (2)

Как предложил Филип Чериан, вы можете проверить раздел «Математика». Тем не менее, я хочу указать на одну вещь о том, как справиться с ограничением.

Я предполагаю, что | к | 0 . Первое, что вы хотите сделать, это разделить каждый пропагатор на реальную и мнимую части. В общем,

лим дельта 0 + 1 Е Е ( д ) + я дельта "=" лим дельта 0 + [ 1 Е Е ( д ) я дельта [ Е Е ( д ) ] 2 + дельта 2 ] ,
где я умножил верх и низ на комплексное сопряжение знаменателя и не принял во внимание дельта 2 в знаменателе действительной части, так как он просто исчезнет, ​​когда вы возьмете предел. Затем вы используете личность
лим дельта 0 + дельта [ Е Е ( д ) ] 2 + дельта 2 "=" π дельта ( Е Е ( д ) ) ,
то есть дельта Дирака. То, что вы получаете, это
г 3 д [ 1 Е Е ( д ) я π дельта ( Е Е ( д ) ) ] [ 1 Е Е ( | д + к | ) я π дельта ( Е Е ( д + к ) ) ] "=" г 3 д 1 Е Е ( д ) 1 Е Е ( | д + к | ) + я π г 3 д [ дельта ( Е Е ( д ) ) Е Е ( | д + к | ) + дельта ( Е Е ( | д + к | ) ) Е Е ( д ) ] .

Обратите внимание, что, поскольку я предполагаю к 0 , произведение двух дельт Дирака должно будет равняться нулю, потому что их аргументы никогда не равны нулю одновременно. Вы можете легко вычислить интеграл с помощью дельт Дирака. Напомним, что, поскольку дисперсия квадратична по д , при изменении переменных необходимо соблюдать определенные правила. Это дает вам мнимую часть интеграла. У меня нет реальных советов, как оценить реальную часть

р е { я ( к ) } "=" г 3 д 1 Е Е ( д ) 1 Е Е ( | д + к | ) ,
кроме проверки того, что вам это действительно нужно. В зависимости от проблемы, которую вы решаете, иногда все, что вам нужно, это воображаемая часть. С другой стороны, я также рекомендую вам проверить, что оба знаменателя в я ( к ) иметь + я ϵ , вместо того, чтобы иметь + я ϵ и другие я ϵ . Я говорю это потому, что это выражение выглядит как скелетное приближение к поляризационному пузырю, который является продуктом запаздывающего и опережающего пропагаторов, и в этом случае их мнимые части имеют противоположные знаки.

Это модификация классических манипуляций, которые делаются при вычислении петлевых интегралов. Используемый инструмент обычно называют «параметрами Фейнмана». Е я и определение м 2 2 Е чтобы установить связь с обычными вычислениями, которые я нахожу (я опускаю ϵ , так как они здесь неуместны):

1 2 я "=" г 3 д 1 ( д 2 м 2 ) ( ( д + к ) 2 м 2 ) "=" 0 1 г Икс г 3 д 1 [ ( ( д + к ) 2 м 2 ) Икс + ( д 2 м 2 ) ( 1 Икс ) ] 2
Немного расширив и переписав выражение,
1 2 я "=" 0 1 г Икс г 3 д 1 [ ( д + к Икс ) 2 к 2 Икс 2 + к 2 Икс м 2 ] 2
Теперь, сдвигая интегральную переменную, я нахожу:
1 2 я "=" 0 1 г Икс г 3 д 1 [ д 2 + Δ ] 2
где Δ к 2 Икс 2 + к 2 Икс м 2 . Угловая часть интеграла теперь тривиальна, а радиальный интеграл прост. Я нахожу,
1 2 я "=" π 2 0 1 г Икс Δ 1 / 2
The Икс интеграл обычно остается нетронутым, но я подозреваю, что в этом случае вы могли бы попробовать выполнить и это.

Отлично, спасибо, это именно то, на что я надеялся. Я приму ответ, как только у меня будет время сесть и поработать над ним самостоятельно. Похоже, что можно будет выполнить Икс интеграл аналитически. Но вообще, что вы подразумеваете под " Икс интеграл обычно не трогают"?
В более сложных петлевых интегралах часто бывает слишком сложно выполнить интегралы по параметрам Фейнмана, поэтому их оставляют как интегралы и при желании выполняют численно.